Convergent Telescoping Series: \sum_{n=1}^{\infty} \arctan(n+1)-\arctan(n)

Click For Summary
The discussion centers on the convergence of the telescoping series \sum_{n=1}^{\infty} \arctan(n+1) - \arctan(n). The limit of the series is evaluated as \lim_{n\rightarrow \infty} (\arctan 2 - \arctan 1) + (\arctan 3 - \arctan 2) + ... + (\arctan(n+1) - \arctan n). Most terms cancel, leaving only \arctan(n+1) and -\arctan(1). The key realization is that as n approaches infinity, \arctan(n+1) approaches \frac{\pi}{2}, leading to the conclusion that the series converges to \frac{\pi}{2} - \arctan(1). The final result confirms the convergence behavior of the series.
courtrigrad
Messages
1,236
Reaction score
2
\sum_{n=1}^{\infty} \arctan(n+1)-\arctan(n).

So we want to take \lim s_{n} = \lim_{n\rightarrow \infty} (\arctan 2-\arctan 1)+(\arctan 3-\arctan 2) + ... + (\arctan(n+1)-\arctan n). From this I can see all of the terms cancel except \arctan 1. But then how do we get: \lim_{n\rightarrow \infty} \arctan(n+1)-\arctan 1? Wouldn't the \arctan(n+1) cancel out?

Thanks
 
Physics news on Phys.org
I got it. I was assuming that it would keep on going. But it stops. I have to take the limit. so \arctan(n+1) and a -\arctan 1 are left.
 
Question: A clock's minute hand has length 4 and its hour hand has length 3. What is the distance between the tips at the moment when it is increasing most rapidly?(Putnam Exam Question) Answer: Making assumption that both the hands moves at constant angular velocities, the answer is ## \sqrt{7} .## But don't you think this assumption is somewhat doubtful and wrong?

Similar threads

  • · Replies 3 ·
Replies
3
Views
1K
  • · Replies 6 ·
Replies
6
Views
2K
Replies
17
Views
2K
  • · Replies 2 ·
Replies
2
Views
2K
  • · Replies 22 ·
Replies
22
Views
910
  • · Replies 7 ·
Replies
7
Views
2K
  • · Replies 2 ·
Replies
2
Views
2K
  • · Replies 5 ·
Replies
5
Views
2K
  • · Replies 3 ·
Replies
3
Views
2K
  • · Replies 17 ·
Replies
17
Views
2K